Difference between revisions of "2021 AIME II Problems/Problem 14"

m (Solution 4 (Easy and Simple): Great solution! :) I made the parentheses larger.)
(Solution 4 (Easy and Simple))
Line 49: Line 49:
  
 
==Solution 4 (Easy and Simple)==
 
==Solution 4 (Easy and Simple)==
Firstly, let <math>M</math> be the midpoint of <math>BC</math>. Then, <math>\angle OMB = 90^o</math>. Now, note that since <math>\angle OGX = \angle XAO = 90^o</math>, quadrilateral <math>AGOX</math> is cyclic. Also, because <math>\angle OMY + \angle OGY = 180^o</math>, <math>OMYG</math> is also cyclic. Now, we define some variables: let <math>\alpha</math> be the constant such that <math>\angle ABC = 13\alpha, \angle ACB = 2\alpha, </math> and <math>\angle XOY = 17\alpha</math>. Also, let <math>\beta = \angle OMG = \angle OYG</math> and <math>\theta = \angle OXG = \angle OAG</math> (due to the fact that <math>AGOX</math> and <math>OMYG</math> are cyclic). Then, <cmath>\angle OXY = 180 - \beta - \theta = 17\alpha \implies \beta + \theta = 180 - 17\alpha.</cmath> Now, because <math>AX</math> is tangent to the circumcircle at <math>A</math>, <math>\angle XAC = \angle CBA = 13\alpha</math>, and <math>\angle CAO = \angle OAX - \angle CAX = 90 - 13\alpha</math>. Finally, notice that <math>\angle AMB = \angle OMB - \angle OMG = 90 - \beta</math>. Then, <cmath>\angle BAM = 180 - \angle ABC - \angle AMB = 180 - 13\alpha - (90 - \beta) = 90 + \beta - 13\alpha.</cmath> Thus, <cmath>\angle BAC = \angle BAM + \angle MAO + \angle OAC = 90 + \beta - 13\alpha + \theta + 90 - 13\alpha = 180 - 26\alpha + (\beta + \theta),</cmath> and <cmath>180 = \angle BAC + 13\alpha + 2\alpha = 180 - 11\alpha + \beta + \theta \implies \beta + \theta = 11\alpha.</cmath> However, from before, <math>\beta+\theta = 180 - 17 \alpha</math>, so <math>11 \alpha = 180 - 17 \alpha \implies 180 = 28 \alpha \implies \alpha = \frac{180}{28}</math>. To finish the problem, we simply compute <cmath>\angle BAC = 180 - 15 \alpha = 180 \cdot \left(1 - \frac{15}{28}\right) = 180 \cdot \frac{13}{28} = \frac{585}{7},</cmath> so our final answer is <math>585+7=\boxed{592}</math>.
+
Firstly, let <math>M</math> be the midpoint of <math>BC</math>. Then, <math>\angle OMB = 90^o</math>. Now, note that since <math>\angle OGX = \angle XAO = 90^o</math>, quadrilateral <math>AGOX</math> is cyclic. Also, because <math>\angle OMY + \angle OGY = 180^o</math>, <math>OMYG</math> is also cyclic. Now, we define some variables: let <math>\alpha</math> be the constant such that <math>\angle ABC = 13\alpha, \angle ACB = 2\alpha, </math> and <math>\angle XOY = 17\alpha</math>. Also, let <math>\beta = \angle OMG = \angle OYG</math> and <math>\theta = \angle OXG = \angle OAG</math> (due to the fact that <math>AGOX</math> and <math>OMYG</math> are cyclic). Then, <cmath>\angle XOY = 180 - \beta - \theta = 17\alpha \implies \beta + \theta = 180 - 17\alpha.</cmath> Now, because <math>AX</math> is tangent to the circumcircle at <math>A</math>, <math>\angle XAC = \angle CBA = 13\alpha</math>, and <math>\angle CAO = \angle OAX - \angle CAX = 90 - 13\alpha</math>. Finally, notice that <math>\angle AMB = \angle OMB - \angle OMG = 90 - \beta</math>. Then, <cmath>\angle BAM = 180 - \angle ABC - \angle AMB = 180 - 13\alpha - (90 - \beta) = 90 + \beta - 13\alpha.</cmath> Thus, <cmath>\angle BAC = \angle BAM + \angle MAO + \angle OAC = 90 + \beta - 13\alpha + \theta + 90 - 13\alpha = 180 - 26\alpha + (\beta + \theta),</cmath> and <cmath>180 = \angle BAC + 13\alpha + 2\alpha = 180 - 11\alpha + \beta + \theta \implies \beta + \theta = 11\alpha.</cmath> However, from before, <math>\beta+\theta = 180 - 17 \alpha</math>, so <math>11 \alpha = 180 - 17 \alpha \implies 180 = 28 \alpha \implies \alpha = \frac{180}{28}</math>. To finish the problem, we simply compute <cmath>\angle BAC = 180 - 15 \alpha = 180 \cdot \left(1 - \frac{15}{28}\right) = 180 \cdot \frac{13}{28} = \frac{585}{7},</cmath> so our final answer is <math>585+7=\boxed{592}</math>.
  
 
~advanture
 
~advanture

Revision as of 09:26, 15 June 2021

Problem

Let $\Delta ABC$ be an acute triangle with circumcenter $O$ and centroid $G$. Let $X$ be the intersection of the line tangent to the circumcircle of $\Delta ABC$ at $A$ and the line perpendicular to $GO$ at $G$. Let $Y$ be the intersection of lines $XG$ and $BC$. Given that the measures of $\angle ABC, \angle BCA,$ and $\angle XOY$ are in the ratio $13 : 2 : 17,$ the degree measure of $\angle BAC$ can be written as $\frac{m}{n},$ where $m$ and $n$ are relatively prime positive integers. Find $m+n$.

Diagram

~MRENTHUSIASM (by Geometry Expressions)

Solution 1

Let $M$ be the midpoint of $BC$. Because $\angle{OAX}=\angle{OGX}=\angle{OGY}=\angle{OMY}=90^o$, $AXOG$ and $OMYG$ are cyclic, so $O$ is the center of the spiral similarity sending $AM$ to $XY$, and $\angle{XOY}=\angle{AOM}$. Because $\angle{AOM}=2\angle{BCA}+\angle{BAC}$, it's easy to get $\frac{585}{7} \implies \boxed{592}$ from here.

~Lcz

Solution 2

In this solution, all angle measures are in degrees.

Let $M$ be the midpoint of $\overline{BC}$ so that $A,G,$ and $M$ are collinear. Let $\angle ABC=13k,\angle BCA=2k$ and $\angle XOY=17k.$

Note that:

  1. Since $\angle OGX = \angle OAX = 90,$ quadrilateral $OGAX$ is cyclic by the Converse of the Inscribed Angle Theorem.

    It follows that $\angle OAG = \angle OXG,$ as they share the same intercepted arc $OG.$

  2. Since $\angle OGY = \angle OMY = 90,$ quadrilateral $OGYM$ is cyclic by the supplementary opposite angles.

    It follows that $\angle OMG = \angle OYG,$ as they share the same intercepted arc $OG.$

Together, we conclude that $\triangle OAM \sim \triangle OXY$ by AA, so $\angle AOM = \angle XOY = 17k.$

Next, we express $\angle BAC$ in terms of $k.$ By angle addition, we have \begin{align*} \angle AOM &= \angle AOB + \angle BOM \\ &= \underbrace{2\angle BCA}_{\substack{\text{Inscribed} \\ \text{Angle} \\ \text{Theorem}}} + \underbrace{\frac12\angle BOC}_{\substack{\text{Perpendicular} \\ \text{Bisector} \\ \text{Property}}} \\ &= 2\angle BCA + \underbrace{\angle BAC}_{\substack{\text{Inscribed} \\ \text{Angle} \\ \text{Theorem}}}. \end{align*} Substituting back gives $17k=2(2k)+\angle BAC,$ from which $\angle BAC=13k.$

For the sum of the interior angles of $\triangle ABC,$ we get \begin{align*} \angle ABC + \angle BCA + \angle BAC &= 180 \\ 13k+2k+13k&=180 \\ 28k&=180 \\ k&=\frac{45}{7}. \end{align*} Finally, we obtain $\angle BAC=13k=\frac{585}{7},$ from which the answer is $585+7=\boxed{592}.$

~Constance-variance (Fundamental Logic)

~MRENTHUSIASM (Reconstruction)

Solution 3 (Guessing in the Last 3 Minutes, Unreliable)

Notice that $\triangle ABC$ looks isosceles, so we assume it's isosceles. Then, let $\angle BAC = \angle ABC = 13x$ and $\angle BCA = 2x.$ Taking the sum of the angles in the triangle gives $28x=180,$ so $13x = \frac{13}{28} \cdot 180 = \frac{585}{7}$ so the answer is $\boxed{592}.$

Solution 4 (Easy and Simple)

Firstly, let $M$ be the midpoint of $BC$. Then, $\angle OMB = 90^o$. Now, note that since $\angle OGX = \angle XAO = 90^o$, quadrilateral $AGOX$ is cyclic. Also, because $\angle OMY + \angle OGY = 180^o$, $OMYG$ is also cyclic. Now, we define some variables: let $\alpha$ be the constant such that $\angle ABC = 13\alpha, \angle ACB = 2\alpha,$ and $\angle XOY = 17\alpha$. Also, let $\beta = \angle OMG = \angle OYG$ and $\theta = \angle OXG = \angle OAG$ (due to the fact that $AGOX$ and $OMYG$ are cyclic). Then, \[\angle XOY = 180 - \beta - \theta = 17\alpha \implies \beta + \theta = 180 - 17\alpha.\] Now, because $AX$ is tangent to the circumcircle at $A$, $\angle XAC = \angle CBA = 13\alpha$, and $\angle CAO = \angle OAX - \angle CAX = 90 - 13\alpha$. Finally, notice that $\angle AMB = \angle OMB - \angle OMG = 90 - \beta$. Then, \[\angle BAM = 180 - \angle ABC - \angle AMB = 180 - 13\alpha - (90 - \beta) = 90 + \beta - 13\alpha.\] Thus, \[\angle BAC = \angle BAM + \angle MAO + \angle OAC = 90 + \beta - 13\alpha + \theta + 90 - 13\alpha = 180 - 26\alpha + (\beta + \theta),\] and \[180 = \angle BAC + 13\alpha + 2\alpha = 180 - 11\alpha + \beta + \theta \implies \beta + \theta = 11\alpha.\] However, from before, $\beta+\theta = 180 - 17 \alpha$, so $11 \alpha = 180 - 17 \alpha \implies 180 = 28 \alpha \implies \alpha = \frac{180}{28}$. To finish the problem, we simply compute \[\angle BAC = 180 - 15 \alpha = 180 \cdot \left(1 - \frac{15}{28}\right) = 180 \cdot \frac{13}{28} = \frac{585}{7},\] so our final answer is $585+7=\boxed{592}$.

~advanture

Video Solution 1

https://www.youtube.com/watch?v=zFH1Z7Ydq1s

Video Solution 2

https://www.youtube.com/watch?v=7Bxr2h4btWo

~Osman Nal

See also

2021 AIME II (ProblemsAnswer KeyResources)
Preceded by
Problem 13
Followed by
Problem 15
1 2 3 4 5 6 7 8 9 10 11 12 13 14 15
All AIME Problems and Solutions

The problems on this page are copyrighted by the Mathematical Association of America's American Mathematics Competitions. AMC logo.png